Summer is a great time to explore cool problems to keep your skills sharp!  Schedule a class today!

G
Topic
First Poster
Last Poster
Cubes and squares
y-is-the-best-_   61
N 2 hours ago by ezpotd
Source: IMO 2019 SL N2
Find all triples $(a, b, c)$ of positive integers such that $a^3 + b^3 + c^3 = (abc)^2$.
61 replies
y-is-the-best-_
Sep 22, 2020
ezpotd
2 hours ago
Chess game challenge
adihaya   21
N 2 hours ago by Mr.Sharkman
Source: 2014 BAMO-12 #5
A chess tournament took place between $2n+1$ players. Every player played every other player once, with no draws. In addition, each player had a numerical rating before the tournament began, with no two players having equal ratings. It turns out there were exactly $k$ games in which the lower-rated player beat the higher-rated player. Prove that there is some player who won no less than $n-\sqrt{2k}$ and no more than $n+\sqrt{2k}$ games.
21 replies
adihaya
Feb 22, 2016
Mr.Sharkman
2 hours ago
[ELMO2] The Multiplication Table
v_Enhance   27
N 2 hours ago by Mr.Sharkman
Source: ELMO 2015, Problem 2 (Shortlist N1)
Let $m$, $n$, and $x$ be positive integers. Prove that \[ \sum_{i = 1}^n \min\left(\left\lfloor \frac{x}{i} \right\rfloor, m \right) = \sum_{i = 1}^m \min\left(\left\lfloor \frac{x}{i} \right\rfloor, n \right). \]
Proposed by Yang Liu
27 replies
v_Enhance
Jun 27, 2015
Mr.Sharkman
2 hours ago
Problem 1
randomusername   74
N 3 hours ago by Mr.Sharkman
Source: IMO 2015, Problem 1
We say that a finite set $\mathcal{S}$ of points in the plane is balanced if, for any two different points $A$ and $B$ in $\mathcal{S}$, there is a point $C$ in $\mathcal{S}$ such that $AC=BC$. We say that $\mathcal{S}$ is centre-free if for any three different points $A$, $B$ and $C$ in $\mathcal{S}$, there is no points $P$ in $\mathcal{S}$ such that $PA=PB=PC$.

(a) Show that for all integers $n\ge 3$, there exists a balanced set consisting of $n$ points.

(b) Determine all integers $n\ge 3$ for which there exists a balanced centre-free set consisting of $n$ points.

Proposed by Netherlands
74 replies
randomusername
Jul 10, 2015
Mr.Sharkman
3 hours ago
Find Triples of Integers
termas   41
N 3 hours ago by ilikemath247365
Source: IMO 2015 problem 2
Find all positive integers $(a,b,c)$ such that
$$ab-c,\quad bc-a,\quad ca-b$$are all powers of $2$.

Proposed by Serbia
41 replies
termas
Jul 10, 2015
ilikemath247365
3 hours ago
DO NOT OVERSLEEP JOHN MACKEY’S CLASS
ike.chen   31
N 3 hours ago by Mr.Sharkman
Source: USA TSTST 2023/4
Let $n\ge 3$ be an integer and let $K_n$ be the complete graph on $n$ vertices. Each edge of $K_n$ is colored either red, green, or blue. Let $A$ denote the number of triangles in $K_n$ with all edges of the same color, and let $B$ denote the number of triangles in $K_n$ with all edges of different colors. Prove
\[ B\le 2A+\frac{n(n-1)}{3}.\](The complete graph on $n$ vertices is the graph on $n$ vertices with $\tbinom n2$ edges, with exactly one edge joining every pair of vertices. A triangle consists of the set of $\tbinom 32=3$ edges between $3$ of these $n$ vertices.)

Proposed by Ankan Bhattacharya
31 replies
ike.chen
Jun 26, 2023
Mr.Sharkman
3 hours ago
Grade IX - Problem I
icx   23
N 4 hours ago by shendrew7
Source: Romanian National Mathematical Olympiad 2007
Let $a, b, c, d \in \mathbb{N^{*}}$ such that the equation \[x^{2}-(a^{2}+b^{2}+c^{2}+d^{2}+1)x+ab+bc+cd+da=0 \] has an integer solution. Prove that the other solution is integer too and both solutions are perfect squares.
23 replies
icx
Apr 13, 2007
shendrew7
4 hours ago
USAMO 2002 Problem 2
MithsApprentice   35
N 4 hours ago by sami1618
Let $ABC$ be a triangle such that
\[ \left( \cot \dfrac{A}{2} \right)^2 + \left( 2\cot \dfrac{B}{2} \right)^2 + \left( 3\cot \dfrac{C}{2} \right)^2 = \left( \dfrac{6s}{7r} \right)^2,  \]
where $s$ and $r$ denote its semiperimeter and its inradius, respectively. Prove that triangle $ABC$ is similar to a triangle $T$ whose side lengths are all positive integers with no common divisors and determine these integers.
35 replies
1 viewing
MithsApprentice
Sep 30, 2005
sami1618
4 hours ago
Center lies on altitude
plagueis   17
N 4 hours ago by bin_sherlo
Source: Mexico National Olympiad 2018 Problem 6
Let $ABC$ be an acute-angled triangle with circumference $\Omega$. Let the angle bisectors of $\angle B$ and $\angle C$ intersect $\Omega$ again at $M$ and $N$. Let $I$ be the intersection point of these angle bisectors. Let $M'$ and $N'$ be the respective reflections of $M$ and $N$ in $AC$ and $AB$. Prove that the center of the circle passing through $I$, $M'$, $N'$ lies on the altitude of triangle $ABC$ from $A$.

Proposed by Victor Domínguez and Ariel García
17 replies
plagueis
Nov 6, 2018
bin_sherlo
4 hours ago
IMO Shortlist 2014 C6
hajimbrak   22
N 4 hours ago by awesomeming327.
We are given an infinite deck of cards, each with a real number on it. For every real number $x$, there is exactly one card in the deck that has $x$ written on it. Now two players draw disjoint sets $A$ and $B$ of $100$ cards each from this deck. We would like to define a rule that declares one of them a winner. This rule should satisfy the following conditions:
1. The winner only depends on the relative order of the $200$ cards: if the cards are laid down in increasing order face down and we are told which card belongs to which player, but not what numbers are written on them, we can still decide the winner.
2. If we write the elements of both sets in increasing order as $A =\{ a_1 , a_2 , \ldots, a_{100} \}$ and $B= \{ b_1 , b_2 , \ldots , b_{100} \}$, and $a_i > b_i$ for all $i$, then $A$ beats $B$.
3. If three players draw three disjoint sets $A, B, C$ from the deck, $A$ beats $B$ and $B$ beats $C$ then $A$ also beats $C$.
How many ways are there to define such a rule? Here, we consider two rules as different if there exist two sets $A$ and $B$ such that $A$ beats $B$ according to one rule, but $B$ beats $A$ according to the other.

Proposed by Ilya Bogdanov, Russia
22 replies
hajimbrak
Jul 11, 2015
awesomeming327.
4 hours ago
Radical Axes and circles
mathprodigy2011   4
N Apr 22, 2025 by spiderman0
Can someone explain how to do this purely geometrically?
4 replies
mathprodigy2011
Apr 22, 2025
spiderman0
Apr 22, 2025
Radical Axes and circles
G H J
G H BBookmark kLocked kLocked NReply
The post below has been deleted. Click to close.
This post has been deleted. Click here to see post.
mathprodigy2011
351 posts
#1
Y by
Can someone explain how to do this purely geometrically?
Attachments:
Z K Y
The post below has been deleted. Click to close.
This post has been deleted. Click here to see post.
SirAppel
881 posts
#2
Y by
$AP$ is the radical axis, so then the intersection with $BC$ must have equal powers with respect to both circles, which only occurs when the segment is bisected
Z K Y
The post below has been deleted. Click to close.
This post has been deleted. Click here to see post.
mathprodigy2011
351 posts
#3
Y by
SirAppel wrote:
$AP$ is the radical axis, so then the intersection with $BC$ must have equal powers with respect to both circles, which only occurs when the segment is bisected

thanks :)
Z K Y
The post below has been deleted. Click to close.
This post has been deleted. Click here to see post.
martianrunner
212 posts
#4
Y by
egmo grind :P
This post has been edited 1 time. Last edited by martianrunner, Apr 22, 2025, 4:33 AM
Z K Y
The post below has been deleted. Click to close.
This post has been deleted. Click here to see post.
spiderman0
11 posts
#5
Y by
Let AP intersect BC at M
By power of point we get $MB^2=MP \cdot MA =MC^2 $
Hence, $MB=MC$
This post has been edited 1 time. Last edited by spiderman0, Apr 22, 2025, 7:53 AM
Reason: Typo
Z K Y
N Quick Reply
G
H
=
a